Fleetstreet är nöjd med hjälpen
Fleetstreet 181
Postad: 2 jun 2021 13:49

Area mellan kurvor

Hej! Jag ha uppgiften "Graferna till de två funktionerna y=2sin(3x) och y=4cos3x bildar tillsammans med y-axeln ett
slutet område i första kvadranten. Bestäm detta områdes area." Jag har kommit fram till et svar och undrar om det stämmer?

Så här gick min uträkning,

Jag började med att rita upp grafen.

Sedan räknade jag ut skärningspunkten där de två kurvorna skär första gången i första kvadranten för att hitta det övre gränsvärdet. 

4cos(3x)=2sin(3x)

2sin(3x)=4cos(3x)

Jag dividerade sedan båda leden med cos(3x),

2sin(3x)/cos(3x)=4cos(3x)/cos(3x)

2sin(3x)/cos(3x)=4

Jag skrev om VL till 2tan(3x) med reglerna för de trigonomiska identiteterna,

2tan(3x)=4

Sedan dividerade jag båda leden med 2,

2tan(3x)/2=4/2

tan(3x)=2

Därefter tog jag arctan i båda leden,

3x=tan^-1(2)

Sedan dividerade jag båda leden med 3,

3x/3=tan^-1(2)/3

x=tan^-1(2)/3

Integralens övre gränsvärdet är alltså tan^-1(2)/3, det undre är 0. Jag skrev in värdena och funktionerna i integralen A=a∫^b ((övre funktion)-(undre funktion))dx,

arctan(2)/3^∫0((4cos(3x))-(2sin(3x)))dx

Därefter bestämde jag den primitiva funktionen för (4cos(3x)-(2sin(3x))

f(x)=(4cos(3x)-(2sin(3x)) har enligt reglerna för primitiva funktioner den primitiva funktionen F(x)=((4sin(3x)/3)-(-2cos(3x)/3))=((4/3sin(3x))-(-2/3cos(3x)))

arctan(2)/3^∫0((4cos(3x)-(2sin(3x))dx=[(4/3sin(3x))-(-2/3cos(3x))]0^arctan(2)/3

Därefter satte jag in det övre gränsvärdet i den primitiva funktionen och subtraherade den med det undre gränsvärdet satt i den primitiva funktionen och beräknade det,

[(4sin(3x)/3)-(-2cos(3x)/3)]0^arctan(2)/3=

=((4/3sin(3(tan^-1(2)/3))-(-2/3cos(3(tan^-1(2)/3))-(4/3sin(3(0))-(-2/3cos(3(0)))=

=(2/3)*(1+√5)

Stämmer det? Ska jag isåfall svara exakt eller inte?

Laguna Online 28602
Postad: 2 jun 2021 13:55

Kan du visa vad du ritade?

Fleetstreet 181
Postad: 2 jun 2021 14:00

absolut!

Laguna Online 28602
Postad: 2 jun 2021 14:22

När jag räknar lite i huvudet får jag ditt värde till ungefär 2,16 och hela rektangeln 4*0,5 är mindre än det, så det kan inte stämma.

Fleetstreet 181
Postad: 2 jun 2021 15:09

Oj, det har du rätt i! Vad har jag gjort fel? Ska jag istället använda mig av två integraler, och subtrahera ena området med det andra? Eller är skärningspunktens x-värde fel? 

Fleetstreet 181
Postad: 2 jun 2021 15:35 Redigerad: 2 jun 2021 15:40

Jag kollade med en integralräknare och fick det till 4/9 så jag måste ha gjort något slarvfel. Jag ska räkna det igen och se

edit: nvm, jag gjorde helt fel när jag beräknade det så 4/9 är inte allt rätt

Fleetstreet 181
Postad: 2 jun 2021 16:21

Jag räknade som jag gjorde i början fast ändrade [(4/3sin(3x))-(-2/3cos(3x))]0^arctan(2)/3 till

[(4/3sin(3x))+(2/3cos(3x))]0^arctan(2)/3 och fick svaret 2/3*(-1+√5)≈0,8

Är det rätt?

Laguna Online 28602
Postad: 2 jun 2021 17:22

Jag får samma svar. 

Fleetstreet 181
Postad: 2 jun 2021 21:57

Tack så mycket för hjälpen! 

Svara Avbryt
Close